what is an angle measuring below 90 degrees called?
Answer:
acute angle
Step-by-step explanation:
what is an angle measuring below 90 degrees called?
The acute angle is the angle that has an amplitude of less than 90°. The obtuse angle, on the other hand, has a width greater than 90°. The right angle measures 90° and its sides are orthogonal.
Answer:
an angle less than 90° is an acute angle
Step-by-step explanation:
an angle less than 90° is an acute angle
an angle equal to 90° is a right angle
an angle greater than 90° but less than 180° is an obtuse angle
an angle greater than 180° is a reflex angle
3 5 Solve forx: 2x + - = 4x 4
js say yes or no and 1 sentence explanation not too big
Answer:
Yeah
Owning a phone --> 198/22 = 9
Not owning a phone ---> 72/8 = 9
Total people ----> 270/30 = 9
Im not sure what ur supposed to do but for all of these when u divide the people that aren't late by the people that are late you get 9 so most probably associated
solve for the missing side
Answer:
7.28011
Step-by-step explanation:
a² + b² = c²
7² = 49
2² = 4
49 + 4 = 53
√53 = 7.28011
Answer:
Step-by-step explanation:
2^2+b^2=7^2
4+b^2=49
b^2=45
b= 6.708
plz help me with these questions tomorrow is my exam and i need to practice them
Answer:
good luck !!!!!!!
hope this help for you !!!!!
in a shipment of 65 vials, only 12 do not have hairline cracks. if you randomly select 2 vials from the shipment, in how many ways can at least one of the 2 selected vials have a hairline crack if the order of selection does not matter? a) 1378 b) 3498 c) 16536 d) 2014 e) 262080 f) none of the above
Out of 65 shipment vials, 12 have no hairline cracks. By, using combination formula, there are 2080 ways to select 2 vials, and 2014 ways to select at least one vial with a hairline crack. The correct option is (d) 2014.
To find the number of ways to select 2 vials from 65 without any restrictions, we can use the combination formula:
nCr = n! / (r! * (n-r)!)
where n is the total number of vials (65), and r is the number of vials to be selected (2).
So the total number of ways to select 2 vials from 65 is:
65C2 = 65! / (2! * (65-2)!) = 2080
To find the number of ways to select 2 vials such that at least one has a hairline crack, we can use the complement rule. That is, we can find the total number of ways to select 2 vials, and then subtract the number of ways to select 2 vials with no hairline cracks.
The number of ways to select 2 vials with no hairline cracks is:
12C2 = 66
So the number of ways to select 2 vials with at least one hairline crack is:
2080 - 66 = 2014
Therefore, the correct answer is (d) 2014.
To know more about Combination formula:
https://brainly.com/question/15062986
#SPJ4
graph the following lines y=2/3 x +4
Answer:
See attached graph.
Step-by-step explanation:
This equation is already in slope-intercept form.
This means the term next to the x is the slope, and the constant being added is the y-intercept.
We know that the y-intercept will be 4 because that’s the constant being added. We also know the slope is \(\frac{2}{3}\).
A slope of \(\frac{2}{3}\) will increase in x by 3, for every increase in y by 2.
This info can help us make the graph.
Hope this helped!
which of the following are potential limitations for a research study? choose all that apply. the sample size is too small the sample statistic is not exactly equal to the population parameter the sample is not representative of the population the wording of survey question influences the response self-selected sample
The sample size is too small, the sample is not representative of the population and the wording of survey question influences the response self-selected sample are potential limitations for a research study.
The sample size being too small can limit the accuracy and precision of the results and reduce the power of statistical tests.
A sample not being representative of the population can result in sampling bias, which can affect the validity of the results.
The wording of a survey question can influence the response, which can result in biased results. A self-selected sample, where participants voluntarily participate in a study, can also result in biased results because only those who are interested or motivated to participate will do so.
To know more on sample size
https://brainly.com/question/24214472
#SPJ4
You have 10 cards, numbered 1 to 10, you just picked 4, what is the probability that the next card will be higher
If there are 10 cards, numbered 1 to 10, and 4 is picked, the probability that the next card will be higher is 2/3.
10 cards, numbered 1 to 10 is present.
After picking up the card 4
For the next card
The sample space is (1, 2, 3, 5, 6, 7, 8, 9, 10) = 9
number of desirable outcomes = (5, 6, 7, 8, 9, 10) = 6
Let event A be the probability of getting the next card higher than 4 is
Probability that the next card will be higher = possible outcome / sample space
P(A) = 6/9
P(A) = 2/3
Therefore, if there are 10 cards, numbered 1 to 10, and 4 is picked, the probability that the next card will be higher is 2/3.
To learn more about probability refer here
https://brainly.com/question/24756209
#SPJ1
A market sells 6 cans of food for overy 7 boxes of food. The market sold a total of 26 cans and boxes today. How
many of each kind did the market soll?
A 6 cans and 20 boxes
B 12 cans and 14 boxes
C 14 cans and 12 boxes
D 20 cans and 6 boxes
Edge 2023
Answer:
B
Step-by-step explanation:
For every 7 boxes the market also sells 6 cans; therefore 13 cans and boxes are being sold.
If 26 cans and boxes were sold in total today, then 26/13 = 2
this means 2 of the 6 cans and 7 boxes;
2x6=12 cans
2x7=14 boxes
Therefore, the market sold 12 cans and 14 boxes today
Answer:
B. 12 cans and 14 boxes
Step-by-step explanation:
To calculate how many cans and boxes of food the market sold, set up and solve a system of equations.
Let "x" be the number of cans of food the market sold.
Let "y" be the number of boxes of food the market sold.
According to the given information, we know that the ratio of cans to boxes is 6 : 7. Therefore:
\(x :y=6:7\)
\(\dfrac{x}{y}=\dfrac{6}{7}\)
\(7x=6y\)
As the total number of cans and boxes sold is 26, this can be expressed as x + y = 26.
Therefore, the system of equations that models the given scenario is:
\(\begin{cases}7x=6y\\x + y = 26\end{cases}\)
To solve the system of equations, use the method of substitution.
Rearrange the second equation to isolate y:
\(y=26-x\)
Substitute this into the first equation to eliminate the term in y, and solve for x:
\(7x=6(26-x)\)
\(7x=156-6x\)
\(7x+6x=156-6x+6x\)
\(13x=156\)
\(13x \div 13=156 \div 13\)
\(x=12\)
Substitute the found value of x into the rearranged equation and solve for y:
\(y=26-12\)
\(y=14\)
Therefore, the market sold 12 cans and 14 boxes of food.
Learn more about systems of equations here:
https://brainly.com/question/28164947
The graph shows g(x), which is a translation of f(x) = x². Write the function rule for g(x).
Answer:
g(x) = \((x + 8)^{2}\)
Step-by-step explanation:
Which statement best defines a circle?
A.
the set of all points in a plane that are the same distance from a given point called the center
B.
the set of all points in a plane that are the same distance from each other surrounding a given point called the center
C.
points in a plane that surround a given point called the center
D.
the set of all points that are the same distance from a given point called the center
The set of all points in a plane that are the same distance from a given point called the center. Therefore, option A is the correct answer.
We need to define the circles.
What is the circle?A circle is a two-dimensional figure formed by a set of points that are at a constant or at a fixed distance (radius) from a fixed point (center) on the plane. The fixed point is called the origin or center of the circle and the fixed distance of the points from the origin is called the radius.
Here is a list of properties of a circle:
A circle is a closed 2D shape that is not a polygon. It has one curved face.Two circles can be called congruent if they have the same radius.Equal chords are always equidistant from the center of the circle.The perpendicular bisector of a chord passes through the center of the circle.When two circles intersect, the line connecting the intersecting points will be perpendicular to the line connecting their center points.The set of all points in a plane that are the same distance from a given point called the center. Therefore, option A is the correct answer.
To learn more about the circle visit:
https://brainly.com/question/11833983.
#SPJ1
Answer this question to get marked as brainliest!!!!
Tickets for a game cost a dollars for adults and c dollars for children the expression 3a+5c represents the cost in dollars for a family for the game what is the cost for the family when an adult ticket is 13 and a child ticket costs 8$ Relatively simple but my brain is just farting thanks !
Since an adult ticket costs $13, a child ticket is $8, and expression 3a+5c indicates the price in dollars for a family to attend the game, the total cost is $79 for the event.
what is expression ?Increasing, partitioning, adding, or reducing are all doable in mathematics. As an example, take this expression: Function is defined, equation, and number The components that compose an algebraic equations are numbers, variables, but rather functions (such as addition, subtraction, multiplication or division etc.) One can differentiate two terms or sentences. The terms "expression" or "algorithmic expression" refer to any detailed estimate that suitable parameter, numbers, or an arithmetic action among both them. For instance, the phrase 4m + 5 consists of the terms 4m but also 5, as well as the number m from the given expression, which are all separated by that of the arithmetic symbol +.
given
expression 3a+5c represents the cost
3a + 5c
= 5 * 8 + 3 * 13
= 39 + 40
= 79
Since an adult ticket costs $13, a child ticket is $8, and expression 3a+5c indicates the price in dollars for a family to attend the game, the total cost is $79 for the event.
To know more about expressions visit :-
brainly.com/question/14083225
#SPJ1
Draw the following utility function and estimate the MRS
u(x,y)=min{x,3y}
u(x,y)=x+2y
The first utility function, u(x,y) = min{x, 3y}, represents a utility function where the individual's utility is determined by the minimum value between x and 3y. The second utility function, u(x,y) = x + 2y, represents a utility function where the individual's utility is determined by the sum of x and 2y.
For the utility function u(x,y) = min{x, 3y}, we can graph it by plotting points on a two-dimensional plane. The graph will consist of two linear segments with a kink point. The first segment has a slope of 3, representing the portion where 3y is the smaller value. The second segment has a slope of 1, representing the portion where x is the smaller value. The kink point is where x and 3y are equal.
To estimate the marginal rate of substitution (MRS) for this utility function, we can take the partial derivatives with respect to x and y. The MRS is the ratio of these partial derivatives, which gives us the rate at which the individual is willing to trade one good for another while keeping utility constant. In this case, the MRS is 1 when x is the smaller value, and it is 3 when 3y is the smaller value.
For the utility function u(x,y) = x + 2y, the graph is a straight line with a slope of 1/2. This means that the individual values both x and y equally in terms of utility. The MRS for this utility function is a constant ratio of 1/2, indicating that the individual is willing to trade x for y at a constant rate of 1 unit of x for 2 units of y to maintain the same level of utility.
learn more about utility function here
https://brainly.com/question/31055643
#SPJ11
What is the solution to 7x + 6y if y = 10 and x = 2?
Answer: First we will substitute the numbers in place of the variables in the equation, and then we will use order of operations to solve.
7x + 6y would be equal to:
7(2) + 6(10)
Now we will perform the multiplication:
7 * 2 = 14
6 * 10 = 60
The equation now looks like:
14 + 60
This is equal to 74. Your answer is therefore 74.
Hi there! Hopefully this helps!
~~~~~~~~~~~~~~~~~~~~~~~~~~~~~~~~~~~~~~~~~~~~~~~~~~~~~
Answer: 74First we need to evaluate the variables into the expression.
7x + 6y = 7(2) + 6(10).
6 x 10 = 60
7 x 2 = 14.
60 + 14 = 74
the similarities and differences between calculating the balance of an account earning simple interest and the balance of an account earning compound interest
The similarity between simple interest and compound interest are the principal amount and interest rate while the major difference is in compound interest, the interest accrued are compounded over time
What is Simple InterestSimple interest is a type of interest calculated only on the principal amount of a loan or deposit, without taking into account any compounding effects. It is computed as the product of the principal amount, the interest rate and the length of time the principal is held.
Compound interest is interest that is calculated not only on the initial principal of a deposit or loan, but also on any accumulated interest from prior periods. This means that the amount of interest earned in each period increases as the principal balance increases over time.
The similarity between calculating the balance of an account earning simple interest and the balance of an account earning compound interest is that both calculations involve multiplying the principal amount by the interest rate.
The difference between calculating the balance of an account earning simple interest and the balance of an account earning compound interest is that the balance of an account earning compound interest is calculated by multiplying the principal amount by (1 + the interest rate) and then raising that number to a power equal to the number of compounding periods, whereas the balance of an account earning simple interest is calculated by multiplying the principal amount by (1 + the interest rate multiplied by the number of time periods).
The formulas are given as
Simple Interest = P(1 + rt)
Compound Interest = P(1 + r/n)^nt
where;
P = Principalr = ratet = timen = number of times compoundedLearn more on simple interest here;
https://brainly.com/question/3575751
#SPJ1
3/4x(1-1/9) pease help
Answer:
x = 0
Step-by-step explanation:
To find the excluded values set the denomometer equal to 04x=0
divide both sides of the equation by 4
x=0
so,
x=0
the overall answer to your question is 0
you bought a new car for $17,896. as you drive off the lot , the car depreciates by 6.3%. what will the value of the car be in 9 years ?
Answer: at the end of year 9 the car value would be $9963,5
Step by step:
Every year you lose 6.30% of the last value, so from the last year's value you take out another 6.3%.
every year the formula is the same, so based on first year it would be:
$17896 * (100%-6.3%) = $17896 * 93.5% = $16768.6
year 1 17896,0 6,30%
year 2 16768,6 6,30%
year 3 15712,1 6,30%
year 4 14722,3 6,30%
year 5 13794,8 6,30%
year 6 12925,7 6,30%
year 7 12111,4 6,30%
year 8 11348,4 6,30%
year 9 10633,4 6,30%
end of year 9 = 9963,5
suppose we want to test whether there is a difference between the room rates of luxury hotels in new york versus los angeles. we collect data from 20 random luxury hotels in new york and 30 random luxury hotels in l.a. what are the degrees of freedom associated with a pooled-variance two-sample t-test or confidence interval? type your answer here
The degrees of freedom associated with a pooled-variance two-sample t-test or confidence interval in the given case would be 48.
When a pooled-variance two-sample t-test or confidence interval is to be conducted on two groups, the degrees of freedom is calculated using the following formula: df = n1 + n2 - 2, where n1 and n2 are the sample sizes of the two groups being compared.
In the given scenario, data is collected from 20 random luxury hotels in New York and 30 random luxury hotels in Los Angeles. Therefore, the degrees of freedom for a pooled-variance two-sample t-test or confidence interval would be:df = 20 + 30 - 2df = 48. Hence, the degrees of freedom associated with a pooled-variance two-sample t-test or confidence interval are 48.
Learn more about degrees of freedom:
https://brainly.com/question/28527491
#SPJ11
What must be the value of c , if the following is to be a probability density function? Round your answer to two decimal places.
{c(5x − 4 − x2)0if 1 ≤ x ⩽ 4otherwise Numeric Response
The value of c that makes the given function a probability density function is 3/2.
A probability density function is a function that describes the likelihood of a random variable taking on a specific value within a given range.
In order for a function to be a probability density function, it must satisfy certain conditions, such as being non-negative and integrating to 1 over its domain.
In this problem, we are given a function:
c(5x - 4 - x²)if 1 ≤ x ≤ 4, and 0 otherwise.
We need to find the value of c that will make this function a probability density function. That means we need to check whether the function is non-negative and integrates to 1 over the interval [1, 4].
First, let's check whether the function is non-negative. Since c is a constant, we just need to look at the expression inside the parentheses.
For this expression to be non-negative, we need to find its roots:5x - 4 - x² = 0⇒ x² - 5x + 4 = 0⇒ (x - 1)(x - 4) = 0The roots are x = 1 and x = 4.
We can see that the expression inside the parentheses is negative between these two roots, and positive outside this interval.
Therefore, the function is only non-negative for values of x between 1 and 4.Next, let's check whether the function integrates to 1 over the interval [1, 4].
We can do this by evaluating the integral:
integral(1, 4, c(5x - 4 - x²)) = 1
We can simplify this expression by pulling the constant c outside the integral and then integrating the expression inside the parentheses:
integral(1, 4, 5x - 4 - x²) = 1
Using the power rule of integration, we get:
[(5/2)x² - 4x - (1/3)x³]1⁴ = 1
Simplifying this expression, we get:
(5/2)(4²) - 4(4) - (1/3)(4³) - (5/2)(1²) + 4(1) + (1/3)(1³) = 1
Solving for c, we get:
c = 3/2
So the value of c that makes the given function a probability density function is 3/2.
For more such questions on probability, click on:
https://brainly.com/question/30390037
#SPJ8
Please help me !! would appreciate
The answers that describe the quadrilateral DEFG area rectangle and parallelogram.
The correct answer choice is option A and B.
What is a quadrilateral?A quadrilateral is a parallelogram, which has opposite sides that are congruent and parallel.
Quadrilateral DEFG
if line DE || FG,
line EF // GD,
DF = EG and
diagonals DF and EG are perpendicular,
then, the quadrilateral is a parallelogram
Hence, the quadrilateral DEFG is a rectangle and parallelogram.
Read more on quadrilaterals:
https://brainly.com/question/23935806
#SPJ1
Simplify the expression
using only positive exponents in the image
Answer:
option2 12x²/y⁹
Step-by-step explanation:
attached the solution below, hope that helps...
Things that have an equal an equal chance of happening are known as what?
Mutually exclusive events are events that have no overlap between them and the sum of the probabilities of all mutually exclusive events is equal to 1.
What is the probability theory?
Probability theory is a branch of mathematics that deals with the study of random phenomena. It provides a mathematical framework for analyzing the likelihood of different outcomes in a given situation, and for making predictions about future events based on past data.
Things that have an equal chance of happening are known as equally likely events or mutually exclusive events. In probability theory, mutually exclusive events are events that cannot happen at the same time; if one event occurs, the other cannot.
In general, mutually exclusive events are events that have no overlap between them and the sum of the probabilities of all mutually exclusive events is equal to 1.
To learn more about probability theory, Visit
https://brainly.com/question/1391215
#SPJ1
If a consumer's budget increases, the budget line shifts to the _______ and a consumer can reach a _______________ level of consumption. A. right; higher B. left; higher C. right; lower D. left; lower
When a consumer's budget increases, the budget line shifts to the right, allowing the consumer to reach a higher level of consumption. Therefore, the correct answer is A. right; higher
When a consumer's budget increases, it means they have more money available to spend on goods and services. This increase in budget causes the budget line, which represents the different combinations of goods a consumer can afford, to shift. The budget line shows the maximum amount of goods that can be purchased with the given budget and prices.
When the budget line shifts to the right, it indicates that the consumer can now afford to purchase more goods than before. This means that the consumer can reach a higher level of consumption because they have more options available to them within their budget constraint. They can either purchase more of the same goods or allocate their budget towards different goods and increase their overall consumption.
Learn more about increases here: https://brainly.com/question/8791755
#SPJ11
The graph shows the number of miles you and a friend run each week while training for a race.
Answer: 4 miles
Step-by-step explanation:
Use integration in cylindrical coordinates in order to compute the volume of: U={(x,y,z):0≤x2+y2≤1,0≤z≤5−x−y}
The volume of the given set using integration in cylindrical coordinates is 12π.
Main part: The given set U can be visualized as a cylinder of radius 1 and height 5, that is truncated by removing a smaller cylinder of the same height and radius 1/2. We will use cylindrical coordinates (ρ,θ,z) to describe points in space. The volume of the set U is given by the integral:
∫∫∫_U〖dV=∫_0^(2π)∫_0^1∫_0^(5-r)ρdzdρdθ
Solution: We can see that z varies from 0 to 5 - x - y. In cylindrical coordinates, x = ρ cos θ and y = ρ sin θ. Therefore, z = 5 - ρ cos θ - ρ sin θ. Since ρ goes from 0 to 1, we have:
∫_0^(2π)∫_0^1∫_0^(5-r)ρdzdρdθ = ∫_0^(2π)∫_0^1∫_0^((5-ρcosθ-ρsinθ))ρdzdρdθ
= ∫_0^(2π)∫_0^1[5ρ - ρ²/2 - ρ² cos θ/2 - ρ² sin θ/2] dρ dθ
= ∫_0^(2π)[5/2 - 1/2 cos θ - 1/2 sin θ] dθ
= 2π [5/2 - 1/2 cos θ - 1/2 sin θ] from 0 to 2π
= π (5 - cos θ - sin θ) from 0 to 2π
= π (5 - (-1) - 0 - (-1) - 0)
= 12 π
Therefore, the volume of the set U is 12 π.
Conclusion: Thus, the volume of the given set using integration in cylindrical coordinates is 12π.
To know more about volume visit
https://brainly.com/question/6071957
#SPJ11
At WHAT interest rate per 1,300$ yield $104 as simple interest in 8 months
Answer:
12%
Step-by-step explanation:
Simple Interest Formula
\(\large \boxed{ \sf I = Prt}\)
where:
I = interestP = principalr = interest rate (in decimal form)t = time (in years)Given:
I = $104P = $1300t = 8 months = 8/12 yearsSubstitute the given values into the formula and solve for r:
\(\implies \sf 104=1300r \left(\dfrac{8}{12}\right)\)
\(\implies \sf \dfrac{104}{1300}=\left(\dfrac{8}{12}\right)r\)
\(\implies \sf r=\dfrac{104 \cdot 12}{1300 \cdot 8}\)
\(\implies \sf r=\dfrac{1248}{10400}\)
\(\implies \sf r=0.12\)
Convert into a percentage by multiplying by 100:
\(\implies \sf r=0.12 \times 100=12 \%\)
Therefore, the interest rate is 12%.
Learn more about simple interest here:
https://brainly.com/question/23861622
https://brainly.com/question/27743947
wsdbfmnsedbznvmsdbn jkcmendz jvmdsbn xzjkv,dmzb cnxrfjklserjdzcfilksdjcfiovk,dsjnifkvsjdxioglmhsrndxjkflgbnesjkdxhbnk
Answer:
0.149> 0.128
Step-by-step explanation:
Hope this helps!
Solve for x. The polygons in each pair are similar.
Answer:
B thak you very mvuhb:)
Step-by-step explanation:
Step-by-step explanation:
(4x - 6 )÷15 = 36÷184x - 6 = 15 × 24x - 6 = 304x = 30 + 6 4x = 36x = 36÷4x = 9MARK ME AS BRAINLIST PLZ FOLLOW ME